Difference between revisions of "2013 AIME I Problems/Problem 15"

(Solution 1)
Line 1: Line 1:
==Problem 15==
+
==Problem ==
 
Let <math>N</math> be the number of ordered triples <math>(A,B,C)</math> of integers satisfying the conditions  
 
Let <math>N</math> be the number of ordered triples <math>(A,B,C)</math> of integers satisfying the conditions  
 
(a) <math>0\le A<B<C\le99</math>,  
 
(a) <math>0\le A<B<C\le99</math>,  

Revision as of 20:14, 24 January 2021

Problem

Let $N$ be the number of ordered triples $(A,B,C)$ of integers satisfying the conditions (a) $0\le A<B<C\le99$, (b) there exist integers $a$, $b$, and $c$, and prime $p$ where $0\le b<a<c<p$, (c) $p$ divides $A-a$, $B-b$, and $C-c$, and (d) each ordered triple $(A,B,C)$ and each ordered triple $(b,a,c)$ form arithmetic sequences. Find $N$.

Solution 1

From condition (d), we have $(A,B,C)=(B-D,B,B+D)$ and $(b,a,c)=(a-d,a,a+d)$. Condition $\text{(c)}$ states that $p\mid B-D-a$, $p | B-a+d$, and $p\mid B+D-a-d$. We subtract the first two to get $p\mid-d-D$, and we do the same for the last two to get $p\mid 2d-D$. We subtract these two to get $p\mid 3d$. So $p\mid 3$ or $p\mid d$. The second case is clearly impossible, because that would make $c=a+d>p$, violating condition $\text{(b)}$. So we have $p\mid 3$, meaning $p=3$. Condition $\text{(b)}$ implies that $(b,a,c)=(0,1,2)$ or $(a,b,c)\in (1,0,2)\rightarrow (-2,0,2)\text{ }(D\equiv 2\text{ mod 3})$. Now we return to condition $\text{(c)}$, which now implies that $(A,B,C)\equiv(-2,0,2)\pmod{3}$. Now, we set $B=3k$ for increasing positive integer values of $k$. $B=0$ yields no solutions. $B=3$ gives $(A,B,C)=(1,3,5)$, giving us $1$ solution. If $B=6$, we get $2$ solutions, $(4,6,8)$ and $(1,6,11)$. Proceeding in the manner, we see that if $B=48$, we get 16 solutions. However, $B=51$ still gives $16$ solutions because $C_\text{max}=2B-1=101>100$. Likewise, $B=54$ gives $15$ solutions. This continues until $B=96$ gives one solution. $B=99$ gives no solution. Thus, $N=1+2+\cdots+16+16+15+\cdots+1=2\cdot\frac{16(17)}{2}=16\cdot 17=\boxed{272}$.

Solution 2

Condition c gives us that $A \equiv a \pmod p$, etc. Condition d then tells us that C and c can be expressed as $2B - A$ and $2a - b$, respectively. However, plugging what we got from condition c into this, we find that $3a \equiv 3b \pmod p$. From there, we branch off into two cases; either $p = 3$, or $a \equiv b \pmod p$. Realize then that the second case leads to a contradiction, due to condition b. Then, $p = 3$ means that $(b,a,c)$ must be $(0, 1, 2)$. The bash from there is pretty similar to what was done in Solution 1. We get $\boxed{272}$. - Spacesam

See also

2013 AIME I (ProblemsAnswer KeyResources)
Preceded by
Problem 14
Followed by
Last Problem
1 2 3 4 5 6 7 8 9 10 11 12 13 14 15
All AIME Problems and Solutions

The problems on this page are copyrighted by the Mathematical Association of America's American Mathematics Competitions. AMC logo.png